User Avatar
danielleiebradley834
Joined
Apr 2025
Subscription
Free
PrepTests ·
PT102.S3.Q21
User Avatar
danielleiebradley834
Sunday, Mar 30

I got this question correct, and I understand the pattern of reasoning to get to the correct answer; however, I do have a question about the approach used during the question stem analysis. I notice this is a 'Most Strongly Supported' question type; however, the tutor in the video keeps stating and implying that this is 'Strengthen' question, and essentially treats it as such. I'm wondering if, moving forward, I can treat the two question types the same, or if there are circumstances where I should not conflate MSS with Strengthen questions.

#help

PrepTests ·
PT131.S4.P3.Q22
User Avatar
danielleiebradley834
Friday, Aug 30 2024

Going through four years worth of comments to feel validated in the fact that this passage absolutely wrecked me.

User Avatar
danielleiebradley834
Thursday, May 29

The phrase 'presumably cogent' in the correct answer choice seems to indicate that the analogy supporting the main conclusion is unsupported, or baseless. In other questions, we've seen arguments presume validity in the form of baseless or unsupported conclusions (i.e. "author takes for granted"). This seems markedly different from other 'presumably cogent' arguments.

Here, we're given a clear example of why the arguer makes the conclusion that one's account of their physical environment is inaccurate. The analogized argument about physical environment is supported by the phrase "since it occurs from one particular perspective." This doesn't seem 'presumably cogent,' as the author does offer some support. I'm still not understanding why this phrase would not be proper grounds to eliminate answer choice 'C.'

PrepTests ·
PT131.S1.Q8
User Avatar
danielleiebradley834
Wednesday, Aug 21 2024

I selected E as the answer choice because I thought if the children were attuned to the fact that they were being given a placebo, it might skew the results and perhaps have the participants react adversely to the substance they were given. The video explanation skimmed over E as an answer choice-- can someone help me out here?

PrepTests ·
PT151.S4.Q25
User Avatar
danielleiebradley834
Friday, Jul 19 2024

I really don't understand why, for seemingly no reason at all, the tutor decides to use shapes here when normally we're instructed to use letters to indicate subjects. It's completely inconsistent with what is normally demonstrated and, honestly, just confusing.

PrepTests ·
PT108.S3.Q9
User Avatar
danielleiebradley834
Tuesday, Nov 12 2024

Correct Answer (D): Presumes, without giving justification, that the CEO's claims are reflected in actual practice.

Wrongly Selected Answer (B): Presumes, without giving justification, that one is not indifferent to something one considers a high priority.

To be honest, I'm not wholly convinced by the tutor's explanation of why B is the wrong answer. The tutor suggests that the very definition of 'priority' excludes 'indifference,' that if you prioritize something, you are not indifferent to it. However, I find that 'high priority' does not necessarily indicate that someone is 'not indifferent.' For instance, it could be the case that the CEOs do think employee welfare is a high priority, but are indifferent to it because they are unaffected by their employee's welfare on a personal level. For instance, perhaps I find that waking up on time for work is a high priority, but I'm indifferent to it because I happen to be a lazy person with little regard for repercussions.

Rather, I think that B can be ruled out because the arguer specifically states that the CEOs do not 'behave indifferently,' yet makes no real claim on whether or not the CEOs are actually indifferent or not. In fact, the CEOs can be incredibly indifferent to employee welfare but, at the same time, see it as a high priority and make actionable policies to encourage employee welfare for other reasons-- optics, increasing productivity, ect., meaning that B could still be true and not be a flaw in the argument.

D is the correct answer because it picks up on the key distinction between 'high priority' and its lack of synonymity with 'behaves indifferently,' which the author incorrectly assumes are connected.

PrepTests ·
PT150.S3.Q11
User Avatar
danielleiebradley834
Wednesday, Feb 12

Once upon a time, I got this question wrong for the same reason most commenters here have.

The shift from the phrase 'ill-advised' in the passage to 'should not' in the answer choice seems like an assumption, something to be cautious about because, as a test-taker, we're cautioned against making assumptions when evaluating question choices. In fact, the tonal difference between 'ill-advised,' which seems neutral, to 'should not,' a clearly declarative stance, is a difficult thing to simply dismiss in the answer selection process.

Reviewing this question a few months later, I think the best way to proceed is through a process of elimination, because the other incorrect answer choices are easier to evaluate.

Answer choice B is a restatement of the premise/contextual information in the first sentence, the framework and conditions which the argument operate under. It simply tells us "hey, this is what is going on with book reviews in newspapers, and now I'm about to make statement about this situation." Definitely not the conclusion.

Answer choices D and E are summarizations of the evidence/support provided in the last sentence, we know this is evidence because it supports the idea that removing book reviews is 'ill-advised.' This is not the conclusion either,

Which leaves answer choice A, which is perhaps a stronger restatement of the conclusion ('this move is ill-advised"), but still the only choice that actually derives its claim from the arguer's main conclusion

PrepTests ·
PT103.S2.Q19
User Avatar
danielleiebradley834
Sunday, Mar 09

I need some help figuring out where I went wrong here.

I selected D because the last sentence seems to present cultural conditioning as a necessary condition for violent behavior. Specifically, the stimulus states: "Human beings in certain situations react to unpleasant stimuli with violence-- but, only because they are conditioned by their culture to react in this manner."

I interpreted this as: "React with Violence → Conditioned by Culture," which is consistent with what the tutor states in the video, that the anthropologist is suggesting that violence is not a genetic phenomenon, but rather a consequence of cultural conditioning (contrapositive: conditioned by culturereact with violence).

Therefore, it followed that D could not be inferred by the passage since it states that "Violent behavior is a product of one's cultural environment," or, "Conditioned by Culture → Violent Behavior;" D therefore, seems to be mistaking the necessary and sufficient conditions from the stimulus.

I see how B is correct; however, I'm not sure how I can explain how D is wrong.

#help

It would have been useful for the tutor to use diagramming or some sort of visual to clarify his explanation, especially since this question uses formal logic signifiers; and, according to 7Sage's own metrics, this is a particularly difficult question.

#feedback

User Avatar

Sunday, Mar 09

danielleiebradley834

Drilling Bugging

Hello! I'm doing a late night study sesh on 7 Sage, and the drilling section keeps refreshing and reloading-- I can't even access the explanation videos right now. Is anyone else having difficulties?

User Avatar
danielleiebradley834
Sunday, Jun 08

I picked the correct answer through process of elimination, but I'm not understanding how answer choice 'C' is indisputably correct.

The arguer does not seem concerned with whether the test is reliable at all, but instead is simply concerned with whether there is broad consensus on the known reliability of that test.

I thought of it like a rule application prompt, almost. In order to be used in Court (for criminal cases), the evidence's reliability must be agreed-upon by the scientific community.

In this case, it seems like this is not flawed, I fail to see how the premises don't support the conclusion. It seems like the argument is ironclad in demonstrating how DNA evidence fails to trigger the conclusion which would allow its usage in Court. Just because DNA evidence is seen as reliable, it still fails to meet the conditions needed for the rule to trigger here.

#help

User Avatar
danielleiebradley834
Sunday, Jun 08

Am I the only one who was skeptical about the author's choice to use public opinion on high taxes generally to support a more narrow application of reducing high taxes?

For instance, no one wants higher income taxes on their own personal income, and may therefore respond to the poll stating 'no' to higher taxes. However, the author seemed to misinterpret this generalized definition of 'taxes' in the poll and apply it to support his conclusion on the narrow scope of 'corporate income tax,' which we were not given insight on.

To me, it seemed like the flaw was between the subject shift from 'high taxes' to 'corporate income tax.' I picked the wrong answer choice, 'C,' for this reason.

Are we just to assume the author is correct to conflate these two things? I'm not seeing how this subject shift is permissible, when in other question types this would be a glaring assumption to bridge.

#help

User Avatar
danielleiebradley834
Thursday, Mar 06

No tell me why I deliberately use the LSAT as an excuse to not date ...

PrepTests ·
PT120.S3.Q8
User Avatar
danielleiebradley834
Monday, Sep 02 2024

I ended up with the correct answer choice here, luckily, despite being both bad at math and bad at car ... stuff? Unlike some of the other people in the comments, I didn't get so far as to label this a numbers/percentage flaw, but rather that the author is deriving a definite conclusion without a full account of any other alternative factors. In my head, I kind of thought "okay, what if the most popular car model just so happened to be the most stolen just from sheer population?" but I wasn't expecting to see this exact line of reasoning in the answer choice. SO I guess I got a bit lucky that the answer choice was a bit more obvious/plain than other flaw answer choices I've run into.

PrepTests ·
PT119.S2.Q10
User Avatar
danielleiebradley834
Monday, Sep 02 2024

This might sound really stupid, but I ruled out 'A' as the correct answer choice incorrectly because it didn't really seem like having an 'indispensable aspect of residency training that requires ... exceptionally long hours' necessarily precludes the conclusion of prohibiting residency physicians from working long hours.

Excuse me if this seems like mental gymnastics (because I'm sure it is), but I thought that if this were the case and residents really did have indispensable aspects of training that required long hours, there is nothing that omits the idea that this training could be revised to not require such long hours...

Like, I kind of thought that even if this were the case, there were ways to reduce the impact of the indispensable aspects of training to not include 80-hour work weeks like lengthening the training calendar in general.

I suppose I see now how 'A' is correct since the LSAT requires that we take things at face value and not make any external assumptions, but seriously. I wouldn't think that a training schedule that requires long hours would necessitate physicians working 80-hour weeks when it could be easily revised.

PrepTests ·
PT120.S1.Q20
User Avatar
danielleiebradley834
Sunday, Sep 01 2024

After looking through some of the prior comments, I think I'm beginning to understand why answer choice 'D' is correct-- however, I initially ruled this out in my answer selection process because the stimulus states that the government should aim to improve the wellbeing of all children; however, answer choice 'D' states that only 'at least some' children would benefit, which led me to believe there is a possibility that only a few children would benefit from subsidized day care. I suppose my mistake here was discrediting a weak but correct answer choice and treating this like a sufficient assumption question rather than what it actually is, a necessary assumption question. Anyone else struggle with the quantifiers here?

PrepTests ·
PT109.S3.Q8
User Avatar
danielleiebradley834
Tuesday, Oct 01 2024

Goddamn it. I got so lost transcribing all the quantifiers and mapping it out that by the time I went back to the answer choices, I started treating it like a sufficient assumption question rather than a must be true question.

Confirm action

Are you sure?